0

Finite and infinite sets - class-IX

Description: finite and infinite sets
Number of Questions: 36
Created by:
Tags: sets set language sets, relations and functions set concepts maths
Attempted 0/32 Correct 0 Score 0

Let $G = {x | x$ is boy of your class$}$ and $H = {y | y$ is a girl of your class$}$. What type of sets G and H are?

  1. Finite sets

  2. Infinite sets

  3. Cannot be determined

  4. None of these


Correct Option: A
Explanation:

Since the number of boys and girls in a class are always finite.
Therefore, set G and H are finite sets.

Classify $C = {..., -3, -2, -1, 0}$as 'finite' or 'infinite'. 

  1. Infinite

  2. Finite

  3. Data insufficient

  4. None of these


Correct Option: A
Explanation:

C is an infinite set as there are many numbers less than $ - 3 $

The set of all animals on the earth is a 

  1. Finite set

  2. Singleton set

  3. Null set

  4. Infinite set


Correct Option: A
Explanation:

Since the numbers of animals on the earth are countable(limited).
Therefore, set of all the animals on the earth is "finite" set.
Option A is correct.

Which of the following sets is non - empty ?

  1. Set of odd natural numbers divisible by 2

  2. ${x: x+4=0, x\in N}$

  3. Set of even prime numbers

  4. ${x:2< x< 3,x\in N}$


Correct Option: C
Explanation:

A non empty set has atleast one element.

From the given options we see that the SET C has one element which is $ 2 $

Which one of the following sets is infinite?

  1. Set of all integers greater than $5$

  2. Set ofall integers between $-10^{10}$ and $+10^{10}$

  3. Set of all prime numbers between $0$ and $10^{100}$

  4. Set of all even prime numbers


Correct Option: A
Explanation:

(2), (3), and (4) are finite sets.

The set of positive integers is ..................

  1. Infinite

  2. Finite

  3. Subset

  4. Empty


Correct Option: A
Explanation:

The set of positive integers is never ending. There is no such defined largest integer. Hence the set is infinite

For any three sets A, B and C, $A \cap (B \cup C)$ is

  1. $(A\cup B)\cup (B\cap C)$

  2. $(A\cap B)\cup (A\cap C)$

  3. $A\cup (B\cap C)$

  4. $(A\cup B)\cap (B\cup C)$


Correct Option: B
Explanation:

In the problem statement we are taking union of $B$ and $C$ and then taking its intersection with $A$.

This means $A\cap (B\cup C)$will contain elements that are in $A$ and are in either $B$ or $C$.
$\therefore A\cap (B\cup C) $ is equivalent to taking intersection of $A,B$ and $A,C$ and then taking there union i.e. $(A\cap B)\cup(A\cap C)$
Hence, option B is correct.

Define finite set.
Is $A=$set of animals on the earth a finite set.

  1. True

  2. False


Correct Option: A
Explanation:

Definition: A finite set is a set which has fixed or finite number of elements.
$A=$ Set of animals on the earth is a finite set because number of animals on the earth is large in number, but it is finite/fixed.

State which of the following are finite sets.
$(i){x:x\in N }$ and $(x-1)(x-2)=0.$
$(ii){x:x\in N }$ and $x$ is prime.
$(ii){x:x\in N }$ and $x$ is odd.

  1. $(i)$ only

  2. $(i),(ii)$ and $(iii)$

  3. $(i)$ and $(ii)$

  4. $(ii)$ and $(iii)$


Correct Option: A
Explanation:

$(i)(x-1)(x-2)=0$

$\Rightarrow x-1=0$ or $x-2=0$
$\Rightarrow x=1$ or $x=2$, Number of solutions of equation are finite.
$(ii){2,3,5,7,11,...}$, This set contains infinte number of elements.
$(iii){..,1,3,5,7,...}$, This set contains infinite number of elements. 

Which of the following sets is not a finite set ?

  1. ${ (x,y):{ x }^{ 2 }+{ y }^{ 2 }\le 1\le x+y,\ \ x,y\in R} $

  2. ${ (x,y):{ x }^{ 2 }+{ y }^{ 2 }\le 1\le x+y,\ \ x,y\in Z} $

  3. ${ (x,y):{ x }^{ 2 }\le y\le |x|,\ \ x,y\in Z} $

  4. ${ (x,y):{ x }^{ 2 }+{ y }^{ 2 }=1,\ \ x,y\in Z} $


Correct Option: A
Explanation:

The set ${ (x,y):{ x }^{ 2 }+{ y }^{ 2 }\le 1\le x+y,\ \ x,y\in R} $ consists of all the points in the first quadrant which lie inside the circle ${ x }^{ 2 }+{ y }^{ 2 }=1$ and above the line $x+y=1$ .So, it is not a finite set.
Option $A$ is correct.

Which of the following is incorrect.

  1. The power set of an infinite set is infinite.

  2. The union of two infinite set is infinite.

  3. The intersection of two infinite set is infinite.

  4. The cardinality of an infinite set is infinite.


Correct Option: C
Explanation:

The intersection of an infinite set may be finite.
Example:
$A={x:x\in N; x>2}$
$B={x:x\in I; x<5}$
Here, Both $A$ and $B$ are infinite but its intersection are finite.

Define infinite set .
Is ${x:x\in R:1\le x\le 3}$ a infinite set?

  1. True

  2. False


Correct Option: A
Explanation:

Definition: A set having infinite number of elements is known as infinite set.
 ${x:x\in R:1\le x\le 3}$ is a infinite set because since $x\in R$, there are infinte number of real numbers lie in between two numbers.

Choose that set of numbers from the option set that is similar to the given set {10,15,65}

  1. ${10, 6, 5}$

  2. ${124, 5, 3}$

  3. ${95, 25, 5}$

  4. ${168, 15, 4}$


Correct Option: C
Explanation:

option C 

since in both the given set all the elements are divisible y 5 

Let S be the set of all values of x such that $log _{2x}(x^{2}+5x+6)<1$ then the sum of all integral value of x in the set S, is

  1. 0

  2. 8

  3. 9

  4. 10


Correct Option: A
Explanation:
$ log _{2x}(x^{2}+5x+6)< 1 $

$ \Rightarrow x^{2}+5x+6< 2x^1 $

$ \Rightarrow x^{2}+3x+6< 0 $

But $ x^{2}+3x+6 = 0 $ has no real roots 

$ \therefore S$  is an empty set 

$ \therefore $ sum of all integral values of $ x = 0 $ 

If a set contains $n$ elements then number of elements in its power set is

  1. $2^n-n$

  2. $2^n-2$

  3. $2^n$

  4. $n^2$


Correct Option: C
Explanation:

Given the set contains $n$ elements.

Then its power set will contains $2^n$ elements.

If $A,B$ are two non-empty sets which of the following statement is false

  1. $A-B=A\cap \left( { B }^{ C } \right) $

  2. $A-B=A\cap \left( { A\cap B } \right) $

  3. $A-B=A-\left( { B }^{ C } \right) $

  4. $A-B=\left( { A\cup B } \right) -B$


Correct Option: C

If $A=\left{1, 2, 3\right}$, then the numbers of subsets of set $A$ containing element $3$, is 

  1. $24$

  2. $28$

  3. $8$

  4. $16$


Correct Option: C
Explanation:
The set $\left\{1, 2, 3\right\}$ has $8$ subsets. The first subset would be the null or empty subset, which contains none of the numbers: $\left\{\right\}$.
 The null set is a subset of every set. The other subsets would include some of the numbers in the set, but not all of them: $\left\{1\right\}$,$\left\{2\right\}$,$\left\{3\right\}$,$\left\{1,2\right\}$,$\left\{1,3\right\}$,$\left\{2,3\right\},\{1,2,3\}$

Let ${ a } _{ 1 },{ a } _{ 2 },{ a } _{ 3 },............{ a } _{ 10 }$ be in G.P. with ${ a } _{ i }>0$ for $i=1,2,....,10$ and $S$ be the set of pairs $(r,k),r\quad k\in N$ ( the set of natural numbers) for which
$\left| { log } _{ e }{ a } _{ 1 }^{ r }{ a } _{ 2 }^{ k }\quad { log } _{ e }{ a } _{ 2 }^{ r }{ a } _{ 3 }^{ k }\quad { log } _{ e }{ a } _{ 3 }^{ r }{ a } _{ 4 }^{ k }\ { log } _{ e }{ a } _{ 4 }^{ r }{ a } _{ 5 }^{ k }\quad { log } _{ e }{ a } _{ 5 }^{ r }{ a } _{ 6 }^{ k }\quad { log } _{ e }{ a } _{ 6 }^{ r }{ a } _{ 7 }^{ k }\ { log } _{ e }{ a } _{ 7 }^{ r }a _{ 8 }^{ k }\quad { log } _{ e }{ a } _{ 8 }^{ r }{ a } _{ 9 }^{ k }\quad { log } _{ e }{ a } _{ 9 }^{ r }{ a } _{ 10 }^{ k } \right| =0$
Then the number of elements in S, is :

  1. Infinitely many

  2. 4

  3. 10

  4. 2


Correct Option: A

Classify $A = {x | x$ is a multiple of $3}$ as 'finite' or 'infinite'.

  1. Finite

  2. Infinite

  3. Ambiguous

  4. Data insufficient


Correct Option: B
Explanation:

A will be an infinite set as we have many multiples of 3 such has 3, 6, 9, 12 and so on

Classify $D = {x | x = 2^n, n \in N}$ as 'finite' or 'infinite'.

  1. Infinite

  2. Finite

  3. Data insufficient

  4. None of these


Correct Option: A
Explanation:

Set D will be infinite as there will be infinite powers of 2 since n can be any natural number from 1 to infinity.

Classify $B = {y | y$ is a factor of $13}$ as 'finite' or 'infinite'.

  1. Infinite

  2. Finite

  3. Data insufficient

  4. None of these


Correct Option: B
Explanation:

B is a finite set as 13 has only two factors which are 1, and 13.

The set of fractions between the natural numbers 3 and 4 is a :

  1. Finite set

  2. Null set

  3. Infinite set

  4. Singleton set


Correct Option: C
Explanation:

We can have many fractions between the numbers 3 and 4.
Hence, this set is an infinite set

If $A$ is finite set. Let $n(A)$ denote the number of elements in $A$ and $B$ are finite sets, $A\neq B$ and $n(A) = n(B)$. Then $n(A\cap B)$ is

  1.  $ > n(A)$

  2.  $ < n(A)$

  3.  $ \neq n(A)$

  4.  $ \leq n(A)$


Correct Option: B

Identify the type of Set
$A= { x| x \epsilon N, 2 \leq x \leq 3}$

  1. Finite Set

  2. Infinite Set

  3. Null Set

  4. Singleton Set


Correct Option: A
Explanation:

We have to identify the type of set.

Given $A={x|x\in N, 2 \leq x \leq 3 }$

               $={2,3}$ which is a finite set.

Therefore $A$ is a finite set.

A finite set $S$ is given by $S={x:x\in N: x\le15}.$ Find the cardinality of its power set.

  1. 32952

  2. 16384

  3. 32768

  4. 16476


Correct Option: C
Explanation:

$S$ is given by $S={x:x\in N: x\le15}$
$S={1,2,3,4,5,6,7,8,9,10,11,12,13,14,15}$
$\therefore n(S)=15$
The power set is set of all the possible subsets of $S.$
Number of elements in power set $=$ Total number of subsets
So, cardinality of power set $S=2^{15}=32768$

Which of the following are countably infinite and uncountably infinte.
$(i)$Set of natural numbers
$(ii)$Set of real numbers

  1. Both $(i)$ and $(ii)$ are uncountably infinite

  2. $(i)$ uncountably infinite and $(ii)$ countably infinite

  3. $(i)$ countably infinite and $(ii)$ uncountably infinite

  4. Both $(i)$ and $(ii)$ are countably infinite


Correct Option: C
Explanation:

Countably Infinite: Set of natural numbers are countably infinite because one can count natural number one after other easily.
Uncountably infinite: Set of real numbers are uncountably infinite because real numbers cannot be counted.

State which of the following are infinite sets.
$(i)A={x:x\in Z: x^2 $ is even $}$
$(ii)B={x:x\in R:-4<x<-2}$

  1. $(i)$ only

  2. $(ii)$ only

  3. $(i)$ and $(ii)$ both

  4. Neither $(i)$ nor $(ii)$


Correct Option: C
Explanation:

$(i)A={x:x\in Z: x^2 $ is even $}$
$A={...,-6,-4,-2,0,2,4,6,...}$ which is an infinite set.
$(ii)B={x:x\in R:-4<x<-2}$
There will be infinite real numbers any two numbers so, it is an infinite set.

Which of the following are infinite set?
$(i)$The set of lines which are parallel to x-axis.
$(ii)$The set of animals living on the earth.
$(iii)$ The set of numbers which are multiple of $5.$
$(iv)$ The set of the circles passing through the origin $(0,0).$

  1. $(i),(ii)$ and $(iv)$

  2. $(ii)$ only

  3. $(i),(iii)$ and $(iv)$

  4. $(i),(ii)$ and $(iii)$$


Correct Option: C
Explanation:

$(i)$The set of lines which are parallel to x-axis is an infinite set because line parallel to x-axis are infinite in number.
$(ii)$The set of animals living on the earth is a finite set because the number of animals living on the earth is finite (although it is quite a big number)
$(iii)$ The set of numbers which are multiple of $5$ is an infinite numbers multiples of $5$ are infinite in number.
$(iv)$ The set of the circles passing through the origin $(0,0)$ is an infinite set because infinite number of circles can pass through the origin.

Which of the following sets are finite sets.
$(i)$ The sets of months in a year.
$(ii){1,2,3,....}$
$(iii){1,2,3,...,99,100}$
$(iv)$ The set of positive integers greater than $100.$ 

  1. $(i)$ and $(iii)$

  2. $(i)$ only

  3. $(ii),(iii)$ and $(iv)$

  4. $(ii)$ and $(iv)$


Correct Option: A
Explanation:

$(i)$ The sets of months in a year is a finite set because it has $12$ elements.
$(ii){1,2,3,....}$ is an infinite set as it has infinite number of elements.
$(iii){1,2,3,...,99,100}$ is a finite set as it has number from $1$ to $100$ which is finite in number.
$(iv)$ The set of positive integers greater than $100$ is an infinite set because positive integers greater than $100$ are infinite in number. 

If $A={a,{b}},$ find $P(A).$

  1. $P(A)={\phi,a,{b},{a,{b}}}$

  2. $P(A)={a,{b},{a,{b}}}$

  3. $P(A)={\phi,{a,{b}}}$

  4. $P(A)={{a,{b}}}$


Correct Option: A
Explanation:

Given that:
$A={a,{b}},$
 $P(A)={\phi,a,{b},{a,{b}}}$

State which of the following are infinite sets.
$(i)A={x:x\in Z: x $ is odd$}$
$(ii)B={x:x\in R:<-10}$

  1. $(i)$ only

  2. $(ii)$ only

  3. $(i)$ and $(ii)$ both

  4. Neither $(i)$ nor $(ii)$


Correct Option: C
Explanation:

$(i)A={x:x\in Z: x^2 $ is even $}$
$A={...,-3,-1-1,3,...}$ which is an infinite set.
$(ii)B={x:x\in R:-2<x<-4}$
$B={...,-14,-13,-12,-11}$ so it is an infinite set.

If the system of equation $x+2y-3z=1$, $(p+2)z=3$, $(2p+1)y+z=2$ has infinite number of solutions, then the value of p is not equal to.

  1. $-2$

  2. $-\displaystyle\frac{1}{2}$

  3. $0$

  4. $2$


Correct Option: A
Explanation:
$x+2y-3z=1$
$(p+2)z=3$
$(2p+1)y+z=2$
let $p=5$, $s\in R/ \left\{ -2, 1/2\right\}$
$\therefore z=\dfrac{3}{s+2}$
$\Rightarrow y=\left(2-\dfrac{3}{s+2}\right)\dfrac{1}{(2s+1)}\Rightarrow \dfrac{2s+1}{(s+2)(2s+1)}-\dfrac{1}{s+2}$
$[As\ 2s+1\neq 0]$
$\therefore x=3z+1-2y$
$=\dfrac{9}{s+2}+1-\dfrac{2}{s+2}=\dfrac{7}{s+2}+1$
$\therefore$ solutions $(x, y, z)=\left(\dfrac{7}{s+2}+1, \dfrac{1}{s+2}, \dfrac{3}{s+2}\right)$
is an infinite set,
$\therefore p$ cannot be equal to $-2$ or $1/2$
- Hide questions